Postmenopausal Sonography & Sonohysterography Flashcards

1
Q

What is the most likely pulsed Doppler characteristic of endometrial
cancer?
A. Low-impedance flow
B. High-impedance flow
C. Absent systolic flow
D. Converse diastolic flow

A

A. Low-impedance flow

How well did you know this?
1
Not at all
2
3
4
5
Perfectly
2
Q

The absence of menstrual bleeding is termed:
A. Amenorrhea
B. Dysmenorrhea
C. Oligomenorrhea
D. Polymenorrhea

A

A. Amenorrhea

How well did you know this?
1
Not at all
2
3
4
5
Perfectly
3
Q

Asherman syndrome is associated with:
A. Uterine leiomyoma
B. Endometrial polyps
C. Endometrial adhesions
D. Ovarian fibroma

A

C. Endometrial adhesions

How well did you know this?
1
Not at all
2
3
4
5
Perfectly
4
Q

What would increase a patient’s likelihood of suffering from
thromboembolism?
A. Polycystic ovary disease
B. ERT
C. Endometrial carcinoma
D. Endometrial atrophy

A

B. ERT

How well did you know this?
1
Not at all
2
3
4
5
Perfectly
5
Q

What is used as a tumor marker for endometrial carcinoma?
A. CR-124
B. CE-125
C. CA-125
D. CA-45

A

C. CA-125

How well did you know this?
1
Not at all
2
3
4
5
Perfectly
6
Q

The removal of tissue from the endometrium by scraping is termed:
A. Dilatation
B. Curettage
C. Sonohysterography
D. Hysteroscopy

A

B. Curettage

How well did you know this?
1
Not at all
2
3
4
5
Perfectly
7
Q

What is the most common form of endometrial carcinoma?
A. Cystadenocarcinoma
B. Krukenberg tumor
C. Adenocarcinoma
D. Squamous cell carcinoma

A

C. Adenocarcinoma

How well did you know this?
1
Not at all
2
3
4
5
Perfectly
8
Q

Measurement of the endometrium should include:
A. The uterine cavity only
B. The deep myometrial echoes and both basal layers
C. The distance from the basal layer to the functional layer
D. The measurement from the basal layer to the basal layer

A

D. The measurement from the basal layer to the basal layer

How well did you know this?
1
Not at all
2
3
4
5
Perfectly
9
Q

The most common cause of postmenopausal bleeding is:
A. Endometrial carcinoma
B. Endometrial atrophy
C. Endometrial leiomyoma
D. Cervical carcinoma

A

B. Endometrial atrophy

How well did you know this?
1
Not at all
2
3
4
5
Perfectly
10
Q

Which of the following is not associated with endometrial hyperplasia?
A. Tamoxifen therapy
B. Polycstic ovary syndrome
C. Ovarian thecoma
D. Asherman syndrome

A

D. Asherman syndrome

How well did you know this?
1
Not at all
2
3
4
5
Perfectly
11
Q

The best description for endometrial polyps is:
A. Malignant nodules that cause bleeding
B. Benign lesions associated with cervical stenosis
C. Malignant nodules that are associated with endometrial atrophy
D. Benign nodules of hyperplastic endometrial tissue

A

D. Benign nodules of hyperplastic endometrial tissue

How well did you know this?
1
Not at all
2
3
4
5
Perfectly
12
Q

Blood accumulation within the uterus is termed:
A. Hematometra
B. Hydrometra
C. Asherman syndrome
D. Endometrial carcinom

A

A. Hematometra

How well did you know this?
1
Not at all
2
3
4
5
Perfectly
13
Q

Which of the following would increase the risk of a patient developing
endometrial cancer?
A. Unopposed ERT
B. Multiparity
C. Osteoporosis
D. Endometrial atrophy

A

A. Unopposed ERT

How well did you know this?
1
Not at all
2
3
4
5
Perfectly
14
Q

What is a gynecologic procedure to remove an endometrial polyp?
A. Hysterectomy with myomectomy
B. Histogram with myomectomy
C. Hysteroscopy with polypectomy
D. Hysteroscopy with polyp myomectomy

A

C. Hysteroscopy with polypectomy

How well did you know this?
1
Not at all
2
3
4
5
Perfectly
15
Q

Cessation of menstruation with advanced age is termed:
A. Asherman disease
B. Premenopausal syndrome
C. Perimenopausal syndrome
D. Menopause

A

D. Menopause

How well did you know this?
1
Not at all
2
3
4
5
Perfectly
16
Q

Stein–Leventhal syndrome is related to all of the following except:
A. Infertility
B. Anovulatory cycles
C. Hirsutism
D. Ovarian hyperstimulation syndrome

A

D. Ovarian hyperstimulation syndrome

How well did you know this?
1
Not at all
2
3
4
5
Perfectly
17
Q

What hormone plays a major role in the symptoms associated with
menopause?
A. hCG
B. LH
C. Estrogen
D. CA-120

A

C. Estrogen

18
Q

The breast cancer treatment drug that may alter the sonographic
appearance of the endometrium is:
A. Progestogen
B. Estrogenate
C. Tamoxifen
D. CA-125

A

C. Tamoxifen

19
Q

Possible benefits of ERT include all of the following except:
A. Reduction in osteoporosis risk
B. Reduction in colon cancer risk
C. Reduction in heart disease risk
D. Reduction in endometrial cancer risk

A

D. Reduction in endometrial cancer risk

20
Q

Which of the following does not occur as a result of menopause?
A. Uterine atrophy
B. Decreased sexual libido
C. Accumulation of fat in the breasts
D. Cystic enlargement of the ovaries

A

D. Cystic enlargement of the ovaries

21
Q

Unopposed estrogen therapy has been shown to increase the risk for
developing:
A. Alzheimer disease
B. Colon cancer
C. Coronary heart disease
D. Endometrial carcinoma

A

D. Endometrial carcinoma

22
Q

The sonographic appearance of a 59-year-old woman on HRT is:
A. Hypoechoic and thickened
B. Hyperechoic and thickened
C. Cystic areas within a thickened endometrium
D. Variable depending upon the menstrual cycle

A

D. Variable depending upon the menstrual cycle

23
Q

Tamoxifen has been linked with all of the following except:
A. Endometrial polyps
B. Endometrial hyperplasia
C. Endometrial leiomyoma
D. Endometrial carcinoma

A

C. Endometrial leiomyoma

24
Q

Which of the following ovarian tumors would be most likely to cause
postmenopausal bleeding?
A. Cystic teratoma
B. Endometrioma
C. Thecoma
D. Fibroma

A

C. Thecoma

25
Q

Tamoxifen effects on the endometrium will sonographically appear as:
A. Cystic changes within a thickened endometrium
B. Cystic areas within a thin endometrium
C. Thin endometrium
D. No apparent effect on endometrial thickness or appearance

A

A. Cystic changes within a thickened endometrium

26
Q

Which of the following would most likely lead to the development of
endometrial adhesions?
A. Endometrial carcinoma
B. D&C
C. Pregnancy
D. Adenomyomatosis

A

B. D&C

27
Q

Causes of postmenopausal bleeding include all of the following except:
A. Asherman syndrome
B. Endometrial atrophy
C. Endometrial hyperplasia
D. Intracavitary fibroids

A

A. Asherman syndrome

28
Q

An asymptomatic 65-year-old patient presents to the sonography
department with pelvic pain but no vaginal bleeding. Her endometrial
thickness should not exceed:
A. 6 mm
B. 8 mm
C. 5 mm
D. 3 mm

A

B. 8 mm

29
Q

An 84-year-old patient presents to the sonography department with
sudden onset of vaginal bleeding. Her endometrium should not exceed:
A. 6 mm
B. 8 mm
C. 5 mm
D. 3 mm

A

C. 5 mm

30
Q

With endometrial atrophy, the endometrial thickness should not exceed:
A. 6 mm
B. 3 mm
C. 8 mm
D. 5 mm

A

D. 5 mm

31
Q

A 68-year-old patient presents to the sonography department complaining
of vaginal bleeding. The most likely cause of her bleeding is:
A. Endometrial carcinoma
B. Endometrial polyps
C. Endometrial atrophy
D. Endometrial fibroids

A

C. Endometrial atrophy

32
Q

A 60-year-old patient presents to the emergency department with sudden
onset of vaginal bleeding. The sonographic examination reveals an
endometrium that measures 4 mm. There are no other significant
sonographic findings. What is the most likely diagnosis?
A. Endometrial atrophy
B. Endometrial carcinoma
C. Endometrial polyp
D. Cervical stenosis

A

C. Endometrial polyp

33
Q

A 67-year-old patient on HRT presents to the sonography department
with abnormal uterine bleeding. Sonographically, the endometrium is
diffusely thickened, contains small cystic areas, and measures 9 mm in
thickness. The most likely cause of her bleeding is:
A. Endometrial atrophy
B. Asherman syndrome
C. Endometrial thecoma
D. Endometrial hyperplasia

A

D. Endometrial hyperplasia

34
Q

Endometrial hyperplasia may be caused by all of the following except:
A. HRT
B. ERT
C. Endometrial atrophy
D. Tamoxifen

A

C. Endometrial atrophy

35
Q

All of the following are clinical findings with endometrial hyperplasia
except:
A. Obesity
B. Polycystic ovary syndrome
C. Abnormal uterine bleeding
D. Thickened endometrium

A

D. Thickened endometrium

36
Q

The sonographic findings of an endometrial polyp may include:
A. Diffuse thickening of the endometrium
B. Menometrorrhagia
C. Intermenstrual bleeding
D. Infertility

A

A. Diffuse thickening of the endometrium

37
Q

Endometrial polyps are associated with all of the following except:
A. Intermenstrual bleeding
B. Tamoxifen therapy
C. Prolapse through the cervix
D. Coronary heart disease

A

D. Coronary heart disease

38
Q

A 34-year-old patient presents to the sonography department for an
endovaginal sonogram complaining of intermenstrual bleeding. The
sonographic findings include a focal irregularity and enlargement of one
area of the endometrium. The most likely diagnosis is:
A. Endometrial polyps
B. Endometrial carcinoma
C. Endometrial atrophy
D. Intramural leiomyoma

A

A. Endometrial polyps

39
Q

The most common female genital tract malignancy is:
A. Ovarian carcinoma
B. Cervical carcinoma
C. Endometrial carcinoma
D. Pelvic inflammatory disease

A

C. Endometrial carcinoma

40
Q

A 31-year-old patient presents to the sonography department for a saline
infusion sonohysterogram complaining of intermenstrual bleeding and
infertility. Sonographically, a mass is demonstrated emanating from the
myometrium and distorting the endometrial cavity. What is the most
likely diagnosis?
A. Endometrial polyp
B. Endometrial carcinoma
C. Endometrial hyperplasia
D. Submucosal leiomyoma

A

D. Submucosal leiomyoma